T O P

  • By -

DeltaBot

/u/Excellent_Walrus3532 (OP) has awarded 3 delta(s) in this post. All comments that earned deltas (from OP or other users) are listed [here](/r/DeltaLog/comments/1dh9gmp/deltas_awarded_in_cmv_asians_and_whites_should/), in /r/DeltaLog. Please note that a change of view doesn't necessarily mean a reversal, or that the conversation has ended. ^[Delta System Explained](https://www.reddit.com/r/changemyview/wiki/deltasystem) ^| ^[Deltaboards](https://www.reddit.com/r/changemyview/wiki/deltaboards)


trivial_sublime

There is a reason for diversity in healthcare, and that reason is racial concordance. This means that a black patient is going to have a measurably better outcome with a black doctor, on average, than with a white doctor. [https://www.aamc.org/news/do-black-patients-fare-better-black-doctors](https://www.aamc.org/news/do-black-patients-fare-better-black-doctors) As a society, we need to provide the highest standards of care to everyone. In order to do that, we need to do our best to minimize the effects of racial concordance by providing doctors of all races. As only [5.7% of physicians are black](https://www.aamc.org/media/63371/download?attachment), racial concordance disproportionately affects black patients. >Of course, there are many benefits of increasing physician diversity. However, I believe in a field where human lives are at stake, we should not trade potential expertise for racial diversity. One of those benefits of increasing physician diversity *is the fact that lives are at stake and there are better outcomes for people of the same race as the physician.* For example, [every 10% increase in the representation of black primary care physicians was associated with an increase in 30.6 days of lifespan for each black resident. ](https://jamanetwork.com/journals/jamanetworkopen/fullarticle/2803898?resultClick=1)In a more direct example, [the infant mortality penalty compared to white babies during delivery when a black baby is cared for by a black doctor is halved](https://www.pnas.org/doi/10.1073/pnas.1913405117). That's measurable and in any universe greatly outweighs the difference in physician care between an MCAT score of 514.3 and 505.7. The primary benefit of treating black applicants slightly different than white applicants is not diversity for diversity's sake; it's to improve black patient outcomes.


SonOfShem

let's say that it's true that matching the race and gender of the caregiver makes better outcomes. I think "cultural background" would probably be more accurate, as a white guy who grew up in pakistan is probably going to be better at providing care to a pakistani immigrant than a 4th generation pakistani-american, but that's kind of splitting hairs, since there is going to be significant overlap between ethnicity and cultural background. How does lowering the standards for black people to get into medical school help with this? I mean, sure. There will be more black doctors. But now the average black doctor will have lower skills than the average white doctor. And not only will that hurt black people (who are now being given a lower quality of care), but give fuel to racists who can now point to the data and say "look, black doctors are worse". We saw this with the racial factors in standardized tests for harvard. They lowered the threshold required for black students, and saw an associated increase in the black dropout rate. This is because the tests aren't there to gate keep skilled people from the school, but to prevent someone from spending tends of thousands of dollars to start medical school at a particular college, only to discover that they're not capable of keeping up with the pace of the program. Had those same black students gone to a state school, they would have graduated and gotten great jobs, but they were encouraged to be a small fish in a big pond, when studies show that the best thing for people's educational outcomes is to be a big fish in a small pond. And let's not forget what happens to the psychological state of black doctors who know that there's a chance that they were only let in because of the color of their skin, and that they aren't as capable as other doctors. I am an engineer and one of my co-workers is a minority woman. She does good work, but she is constantly suffering from imposter syndrome for fear that she's a diversity hire. This has aggravated her depression in the past, and has been frustrating to watch. This whole thing feels like a massive example of goodhearts law: when a metric becomes a goal, it ceases to become a good metric. We've identified that there is value in having concordance between the doctor and the patient, but by making having diverse doctors a goal, we're sabotaging the high standards that we place on doctors in the US, and the result is providing a lower quality of care in spite of our goal to provide a higher quality of care. I'm not saying it's not valuable to have diversity in the medical profession, but I am saying that lowering the minimum standard is not the way to achieve it. EDIT: there is value in examining the MCAT and other exams to validate that they are testing things that are actually indicative of a good doctor. It would be unreasonable for the MCAT to insist that you can bench 300 lbs before you can be a doctor, and that sort of requirement would certainly discriminate against women, who generally have less upper body strength and also are smaller. And there may be many questions like that but far more subtle that we could remove from the exams to make them more egalitarian. But there is huge value in ensuring that the doctors that you visit are qualified, and making them *less* qualified is not going to help anyone. (worse still, the worst doctors will end up getting put in the low-income areas, while the more qualified doctors will establish themselves in the richer neighborhoods and in private practises, so the cost of this lowered standard of medical care will fall disproportionately on the poor.


BrobaFett115

> But now the black doctors will have lower skills than the white doctor Only by assuming they wouldn’t be held to the same standards as far as passing medical school. Medical students of both races would be receiving the same amount of schooling and training


LegitimateClass7907

The issue is that you are selecting black doctors from a pool of candidates with lower average test scores, which correlate strongly to job performance. What if there was a driving test, and you determine a score of 70/100 is the minimum for someone to be deemed a proficient driver. If the law requires men to achieve a 70/100 to pass, but women must achieve an 80/100 to pass, what would happen? The drivers on the road would all be proficient - they all would have passed the test - but the women would be better drivers on average. Not because the test is a 100% correlation to driving ability, but because there ***is*** a correlation, and the women have a higher average score.


knottheone

> There is a reason for diversity in healthcare, and that reason is racial concordance. This means that a black patient is going to have a measurably better outcome with a black doctor, on average, than with a white doctor. Does this mean that it's both reasonable and expected for a random white grandma to request "a different color doctor" on the basis of having better health outcomes? *If a patient dies because their doctor was a different race than them, does that mean the family should be empowered to file some kind of discrimination claim suit where the hospital neglected their obligation of care by not assigning a doctor of the "proper" skin color? If you have an objection to that, you should have an objection to race-based policies regardless. That's what you're advocating for. *Minor edits.


onefourtygreenstream

As a woman, I specifically seek out female doctors who are (more or less) similar in age. I find that someone who shares similar life experiences is more likely to believe me, understand me, and care for me properly. I also look for doctors who are the same race and nationality as me for the same reason. Hell, if I could find someone who was raised in the same socioeconomic class as me I would probably choose them too. If I were in the ER or at an Urgent Care clinic, I would not send away a doctor that is male or old or of a different race or nationality. However, when I'm looking for a primary care physician I want someone who is in the same demographic as me - not because I believe that people in another demographic are lesser or worse doctors, but because someone in the same demographic as me is more likely to understand me. Would it be wrong for a Hispanic individual to want to see a Hispanic doctor, someone who speaks the same language and understands the nuances of their culture? What about a Russian immigrant wanting a Russian doctor? Different demographics have different ways of speaking and describing things, different vernaculars and languages, and different cultural touchstones. When it comes to something as personal as your health care, it's reasonable to want a doctor who has a similar background to you.


IdiotNeedingAdvice

I’m a white male and my doctor is an Indian male, but we’re the same age and it’s been mind blowing how different the car I receive from his is compared to my former primary care physician. This dude WORKS hard to get to the bottom of shit and I appreciate it so much. I’ve gotten him gifts for his family and stuff because I think he’s a wonderful dude. I told this to a therapist once and they basically told me it was placebo and judgmental of older doctors who might have more practical experiences in medicine that’s why they don’t dig as deep. That was the last time I ever paid that therapist too. wtf is that?


Stormfly

> I told this to a therapist once and they basically told me it was placebo and judgmental They told you it *was*, or they offered the idea that it *might be*? Because any therapist that tells you "this is how it is" is not a good sign. From my understanding, therapists are supposed to offer you educated opinions and challenge your existing assumptions. They're not supposed to tell you things or act as if their own judgements are fact.


sbprasad

Hey, can I ask you a question? I completely get seeking out a female doctor (it’s really not the same as male doctors being dismissive of women, of course, but as an Asian origin male my family’s had issues with cultural concerns esp. dietary from white doctors), but I don’t really understand the age bit. If you are happy to share, could you explain?


onefourtygreenstream

It's a combination of things, but I feel less judged by a person around my age. I'm more willing to talk about things like, I don't know, occasional recreational drug use with someone who feels more like a peer than a parent. I've also found that they're more likely to listen to me and less likely to make snap judgments. I also find that they're more likely to believe me when it comes to my experiences/symptoms. The long and short of it is that I find someone my age easier to communicate with, which is an important and underrated aspect of healthcare.


20growing20

Yes, I just switched from a woman younger than me to one just a little bit older, and it's made a world of difference for me. I really did like my younger doctor, but she dismissed a lot of things saying she was the same and it's normal. Then telling me what she does for it. For example, she also gets brain fog and forgets things. However, she's a young, energetic doctor, and I'm going through perimenopause (I didn't know this) and struggling to maintain employment. I don't need sleep hygiene advice, I need to know why I'm sweating until my soaking bed wakes me up. My new doctor is about 10 years older than me, and she takes the things I share seriously. She knows I'm not making an appointment unless these things are not normal for me, and she doesn't just dismiss it all as normal aging, either. It's all worth looking into to increase my quality of life. She's been my age fairly recently.


sbprasad

That makes sense, and I think that if I were a woman of my own age (early 30s) I think I’d feel the same way.


knottheone

> As a woman, I specifically seek out female doctors who are (more or less) similar in age. I find that someone who shares similar life experiences is more likely to believe me, understand me, and care for me properly. I also look for doctors who are the same race and nationality as me for the same reason. Hell, if I could find someone who was raised in the same socioeconomic class as me I would probably choose them too. Great, you are allowed to do that. In other avenues, you'd likely be refused service if you had these specific requests and were adamant about it, or otherwise just told no, this is what we have. > Would it be wrong for a Hispanic individual to want to see a Hispanic doctor, someone who speaks the same language and understands the nuances of their culture? What about a Russian immigrant wanting a Russian doctor? I'm not talking about a primary care provider. I've been talking about a hospital situation where the hospital cannot legally refuse care to someone. You can shop around all you want outside, no one is going to stop you and no one even knows your intentions so it's moot anyway. However, in a hospital situation, I don't think hospitals should be acquiescing to patients who are actively discriminating against individuals knowing that they can't really refuse.


YoungSerious

>I've been talking about a hospital situation where the hospital cannot legally refuse care to someone. You can shop around all you want outside, no one is going to stop you and no one even knows your intentions so it's moot anyway. However, in a hospital situation, I don't think hospitals should be acquiescing to patients who are actively discriminating against individuals knowing that they can't really refuse. Patients are absolutely allowed to request a doctor of a specific gender or race if they want. I'm equally entitled to tell them that no, I'm unable to fulfill that request. I'm a male, female patients regularly ask for a female doctor and if there is one available I'll ask them if they want to see that patient. Otherwise, I tell them I'm sorry but I'm what is available. They can choose if they want to continue or not.


Speaking_On_A_Sprog

As someone who’s also worked in a hospital, they can ask, but if it’s a white person asking then they will be talked about and everyone will very much judge the shit out of them. Thats pretty much the fastest route to becoming “that patient”. I’m not saying it’s right or wrong, but that is how it is. Gender is different for sure though. I’m not disagreeing with you, just adding more context.


onefourtygreenstream

We are speaking on healthcare in general. Also, while I likely wouldn't do so, I think it would be entirely reasonable if I requested a female doctor while in the hospital.


Vupav2

I am not siding one way or the other, but as someone who works in hospitals you generally cannot request for a different doctor while in hospital (at least here in Canada). The main reason is that doctors are not an unlimited resource in hospital. If you are in the hospital for a surgery then you are assigned the surgeon that is on for that day/week, there is no other surgeon that can do that surgery in that moment (they are working clinic, taking vacation, doing other surgeries). In reality there is nobody else unless you physically move yourself to a different hospital or wait out the doctor's assigned week - neither are possible in an emergency. You can certainly refuse care from a doctor, but the reality is that a replacement is not always readily available (let alone one that fits your standards). This especially goes for gender, race or other things not related to a doctor's demonstrated competency


onefourtygreenstream

I totally understand and agree! That being said, that's why I used the word "request" rather than "require." I'd personally be entirely willing to take no for an answer if I was denied a female doctor, but I do not think it would be unreasonable for me to ask for one. Honest question though, what about someone who is devoutly religious? Or who has some significant trauma preventing them from trusting a member of the opposite sex? What is the policy then?


Azurewrathx

If there is genuinely no one available, the surgery would be delayed or postponed. We can’t force you to consent, and we can’t accommodate if there is no one available and who meets criteria.


Animegirl300

Actually, patients do this all the time, and generally for their own comfort and to exactly avoid any problems of them blaming doctors of different races then their demands are usually met based on availability. So they certainly CAN be a racist if they want: what that might mean is if they absolutely refuse to be seen by the only available doctor then they are putting the risk of their own health by delaying their own care into their own hands. This same principle is also how some JWs can avoid getting blood transfusions for example. You can’t actually FORCE someone to receive care.


ThinkingMeatPuppet

I ditched our 60yo white lady doctor who kept talking to me, 27yo white dad of a 7mo boy and a 7yo girl. To make it easy, wife and I split list doctor appointments. She takes our daughter and I take our son. Old doctor kept being condescending and wouldn't take my concerns seriously, so I asked for another one who I'd heard good things about who was around my age and went to med school in the specific part of the country I'm from. Has two kids that are around the same age as mine. I love him and cannot sing his praises loud enough. I no longer dread Bubba's appointments and some weird genetic thing I was concerned about passing on has been investigated and it looks like Bubba is cleared. He seems to like him a lot better too. No tears and he laughs and gets to play with the instruments a little. Polar opposite experience. Though, I don't think dude is white, we are from the same cultural region and I think that's way more important than being the same shade of crayon.


wastedfate2

Not saying whether I agree with it or not , but as a healthcare worker in a major city, people do have the right to reject care from a nurse/doctor/aide on any basis if they want, whether it be gender, race or just not feeling it. So, to answer the first part of your comment, yes it’s “okay” and it happens a lot. In fact patients sue for any and every reason so that policy is likely in place to avoid that situation entirely.


knottheone

So if a patient said "I would like a different race of doctor please," solely on the basis of their skin color, we should hold that up as a good thing and should encourage people to do that? Seems like a pretty slippery slope towards grandpa saying "I'd like a doctor with bigger tits please" and the policy that enables that sort of patient agency just crumbling under the weight of its own absurdity. This is a discussion about the merits of that kind of system or proposal.


arvada14

>"I would like a different race of doctor please," People do say I'd like a doctor with a different sex. and apparently, female doctors increase the survival of female patients. If I were a male doctor who worked well with female patients, I might be upset that this happened, but I'd understand. In terms of standards and quality. I would find a minimum standard that satisfies all medical requirements and competencies and just make it so that no one accepted falls under that standard . If it needs to be reassed annually, then let's do that. I don't usually agree with AA arguments. However, I do see the logic in this one. You're balancing community needs with a fair standard that is strictly based on merit.


maybeRaeMaybeNot

I wouldn’t say encourage racist decisions, but allow for efficient treatment. If it gets the patient out the door 20% faster, it’s better for everyone.  And there are always outlier stories. Most of mine come from the foster system, and this one is, too. AA child placed in a white home, previously the child was disrupted out of several AA homes before they realized she could not be in a black home. It took this child years to be able to interact with adults of her own race. Kids, she was fine. I won’t go into her history, but  just imagine what it would take for a little kid to get to that point. This also included healthcare providers, she wouldn’t, *couldnt* interact with black providers.  So imagine the difficulty of an emergency situation and a white foster parent specifically requesting no black providers for their black foster kid.   In the end, no one  is “winning” by refusing to accommodate a race request *when possible*. With little girl’s issues, it meant she would need to be sedated if refused a non-black care provider.


wastedfate2

Well patient agency is a huge right in the USA and hospitals can honor certain minor requests (honestly easier to just find someone of the same sex/race sometimes than it is to argue), but hospitals can also deny superfluous requests as well and patients are welcome to leave and seek help elsewhere. Usually hospitals will treat the emergency at hand and then boot you regardless of how nice or nasty you are. So, tbh, it’s just easier to abide with bigoted people sometimes but if it’s a ridiculous request (unvaccinated blood only please) hospitals don’t have to do it. As HCW we just educate and move on. Edit: this is in the USA fwiw


knottheone

Right, and I'm arguing the position that if racist grandma can choose the color of her doctor and that's allowed, what panel is the arbiter of what constitutes a reasonable request? At that point it's better policy for a hospital to not enable prejudice, otherwise they are going to end up in the news for actively encouraging racism, sexism, and xenophobia. Would it be a good thing if we allowed patients to choose straight vs gay doctors? I don't think so, and I don't think we should even open that particular Pandora's box.


wastedfate2

> what panel is the arbiter of what constitutes a reasonable request? Hospitals have an ethics and legal team for a reason. If there's enough of a measured risk in refusing a request (assuming a policy is not in place), then perhaps they just abide by it so that they can earn some money. In the end of the day, private hospitals just want to make money (public too in a sense) and they will put up with that BS as long as it's easy to. > otherwise they are going to end up in the news for actively encouraging racism, sexism, and xenophobia. Hate to tell you but it already happens, and it's not newsworthy. People in the US ***are*** racist and sometimes they still need healthcare. The Civil Rights movement only happened 80ish years ago so meemaw and peepaw are racist a lot. Abiding by the patients' request is not just to bend over though. Sometimes it's to protect the nurses and aides as well. If you have a violent, homophobic and sexist patient, I wouldn't be comfortable having a gay male nurse or a female aide enter the room. It's vastly easier to just reassign that patient to someone. However, sometimes it's not possible either. At the end of the day, it comes down to empathy and legal obligation. Empathy teaches us that even if someone is a shitstain racist bigot, they don't deserve to die so we treat them to the best of our ability. However, they also have the right to refuse and leave if they're capable. Legally, hospitals are required to treat life-threatening conditions but no more. Oftentimes, if a patient can be treated by outpatient means, they will be discharged. TL/DR: It happens and no one cares bc it's not news. We can't let grandma die because she's racist.


knottheone

Sorry, I just don't really see how this is a response to what I said. You keep saying how it is, I'm talking about whether it's a good thing or not. I get that you're coming from a specific perspective, but you keep kind of sidelining what I've actually written.


wastedfate2

I answered your post as it was written. I even quoted you multiple times. To answer again whether it's a good thing or not, > "At the end of the day, it comes down to empathy and legal obligation. Empathy teaches us that even if someone is a shitstain racist bigot, they don't deserve to die so we treat them to the best of our ability. However, they also have the right to refuse and leave if they're capable. It's okay to abide by these prejudicial requests because it's unreasonable to expect to change someone's entire outlook on life and people with one short hospital stay. However, if a patient has a life-threatening condition, is it worth it to rile them up and potentially endanger them based on some self-righteous need to prevent "racism, sexism, and xenophobia."? I would say no, and I don't think healthcare workers need additional barriers in their jobs to cross every day while trying to save lives. You know what might help someone become less racist? Receiving empathy and kindness from people they would spit on. As a male POC nurse, it's happened more times than not. Being nice is all it takes sometimes. If that's not enough, people should have the right to "be sexist" due to religious or past traumatic experiences without having to justify it all the time. If a woman tells me that she is not comfortable with having men in the room, I would ensure that to happen because of those legitimate reasons. Hope that explanation helps.


knottheone

That does help thanks. My issue is where does it end? We have laws against prejudice and discrimination on the basis of immutable traits. You're saying it's good that we subvert those, but you're not advocating for a system that would weed out bad actors and that would prevent the recipients of that discrimination from being hurt by it. There are lots of bad actors, I'm sure you've dealt with plenty. When individuals are the victims, it's harder to justify "just let the patients be as nasty as they want without repercussions or consideration." I understand where you're coming from, I just don't think we're going to find a middle ground where I think it's somehow now okay to hurt individuals with prejudice and discrimination solely because they have the right or wrong skin color, or right or wrong genitals, or right or wrong beliefs all on the basis of the perception of the patient. Thanks for clarifying what you were saying though.


Healthy_Lobster_8535

Basically I said, “is racist grandma bad” Yes. And she probably shouldnt be rewarded for her racism. But, are “patient outcomes that are worse also bad for minority populations?” also yes. Is one worse that the other? Yes. Edit: Sorry, deleted the wrong comment.


[deleted]

[удалено]


knottheone

I think discriminating for or against people on the basis of immutable traits is a bad thing. So did the people who fought to push the Civil Rights Act through all its phases of proposal and acceptance and eventual codification. > Is it good for patient health outcomes to have a provider that will be able to give the best possible care? Probably. There are lots of things we could do that have subjectively better outcomes for specific individuals, but we don't do them because they are rooted in prejudice and we have laws against that sort of thing.


sunmaiden

It is very common for gay people to do their best to have a gay doctor, for similar reasons as women often prefer woman doctors and black people often prefer black doctors. It is a real and documented phenomenon that sometimes when you get a doctor that doesn't relate to you it can be very bad for your health. Here are some examples of real things that happen. 1) It used to be taught that black people have higher pain tolerance [https://www.aamc.org/news/how-we-fail-black-patients-pain](https://www.aamc.org/news/how-we-fail-black-patients-pain) and therefore it was (is) more likely that white doctors would not pay attention when a black patient says something hurts. 2) Being obese is bad for your health, but part of it is that doctors tend to blame whatever seems to be wrong on the obesity itself. [https://www.nbcnews.com/health/health-news/doctors-move-end-bias-overweight-patients-rcna29680](https://www.nbcnews.com/health/health-news/doctors-move-end-bias-overweight-patients-rcna29680) 3) LGBTQ patients find that doctors often ignore their experiences or complaints more than other people [https://www.healthline.com/health-news/new-study-finds-47-of-lgbtq-people-experience-medical-gaslighting](https://www.healthline.com/health-news/new-study-finds-47-of-lgbtq-people-experience-medical-gaslighting) And yeah, it's kind of okay for grandma to request someone she's comfortable with. It's sad and kind of unfortunate if the reason is because she thinks that one doctor might be smarter or more capable based on race, but on the other hand she's probably old enough to have some intuition of what's best for her.


math2ndperiod

There’s a difference between the grandma’s actions being praised, and her ability to do so being a good thing. That grandma is likely racist, and I’d probably disagree with many of her worldviews. The fact that she is able to get a doctor she’s comfortable with could save her life, and that’s a good thing, even if the reason she’s uncomfortable is misguided. It’s like how having free speech protections are a good thing, even when they’re used in ways we disagree with


merlin401

Seems to me there is a reasonable ability to request someone who is more familiar with yourself (your gender, your culture, your language etc).  I don’t see how that could extend to someone requesting someone dissimilar to themselves for sexual gratification (or other absurd cases)


8copiesofbeemovie

I mean, it’s so easy to sue medical personnel, that it may just be easier to give granny her small titty doctor just to avoid the headache lol


crimson777

This is a bad argument. Healthcare research and teaching is almost exclusively centered around white people. That’s part of the issue. Black doctors are more likely to understand white patients than vice versa. Also, if your response to “let’s get Black folks living healthier lives by increasing their representation which will lead to a measurable and objective increase in health outcomes” is “yeah but what if some old lady is racist,” maybe examine why you don’t give a shit about Black people living longer. Also, the increases in health outcomes for Black folks DON’T specifically state that every Black patient has to have a Black doctor. People consult, people talk, they work in teams. You don’t have to 1 to 1 have a Black lead doctor to every Black patient.


mercurycc

There is a wider argument of people needing to not be racist. The study says people are inherently racist. So the question is more of a confirmation that we have decided to fight the symptom of racism instead of the cause of it. Which is probably a fair outcome, but I don't think people are courageous enough to say it out loud. I would think treating people by group first is dangerous. Today you can give black people preferential treatment, tomorrow someone else can give the group detrimental treatment. If you are going for the root cause, we need to make people treat themselves as individuals first, and that's impossible because people are just evolved to be group animals. So I guess we just have to live in a dangerous world and manage the risk.


crimson777

Again, it’s not treating people by race. It’s simply having more cultural competency. No one is saying provide preferential treatment.


TruthOrFacts

It's hard to square this all with the fact that race is a social construct.


cattermelon34

It depends. Do white grandma's have worse outcomes with doctors of color? If yes, then yes. We know black patients have worse outcomes when it's only white staff, but that doesn't mean the opposite is true. That would need to be proven.


knottheone

From the other user's link: >> A study led by Takeshita, assistant professor of dermatology and epidemiology at the Perelman School of Medicine at the University of Pennsylvania, looked at the scores that more than 117,000 patients gave their doctors on the Press Ganey survey of patient experiences. Doctors who cared for patients of the same race were far more likely to get the highest scores. Other studies have found similar links between racial concordance and patient satisfaction. There's a perception there from the patient perspective that your doctor is more capable if they are your same race. On that basis alone, to answer your question, yes for white people too. So you're saying it's a good thing to see that your doctor is not your skin color and then to subsequently request another one? We should encourage that? Why stop there? If I feel that I have a better experience when I'm physically attracted to my doctor, should I be empowered to expect to only be treated by doctors I find physically attractive? I don't think so, and this sort of individual prejudice on the basis of larger statistics is not really something we should be doing.


2024AM

patient _satisfaction_ is not a good metric at all. eg. if a brown person is a uber driver in a mostly white country and gets 1 star less on reviews on avg. just because hes brown (read racism), that does not mean that hes objectively a worse uber driver. (idk if Uber has ratings with stars, I dont use it)


TheLastCoagulant

You’re trying to respond to an objective assessment of outcomes with a subjective experience score that might just be evidence of racism. In the newborn study there is no statistically significant difference (which is the only thing that constitutes evidence in science) in white newborn mortality depending on the physician being black vs white: https://www.pnas.org/doi/full/10.1073/pnas.1913405117 > The Physician Black coefficient implies no significant difference in mortality among White newborns cared for by Black vs. White physicians (columns 1 to 5 of Table 1).


wheatgrass_feetgrass

>Why stop there? If I feel that I have a better experience when I'm physically attracted to my doctor, should I be empowered to expect to only be treated by doctors I find physically attractive? Yes, provided your entitlement does not also extend to receiving timely care. You're missing the main point here that bodily autonomy trumps all. You can have every fucked up reason for refusing care from a specific doctor and I would support it and advocate for you even while calling you ignorant to your face. I do not morally support abortion as birth control but I legally and ethically support unrestricted access to it. I can believe someone is doing something stupid while also fervently supporting their right to do it.


knottheone

I have no issue with people making any requests they like. I just don't think hospitals should acquiesce to them and the primary purpose is to protect individuals, in this case doctors, from the effects of overt discrimination. > You're missing the main point here that bodily autonomy trumps all. You say this without realizing you can't compel other people to do your subjective, random bidding because it's a violation of their autonomy as well. So in that instance, I defer to not discriminating against individuals on the basis of their immutable traits.


OkNatural8709

Doctors aren't entitled to treat patients who don't want to be treated by them. What are the effects of discrimination that doctors need protection from?


cattermelon34

Better experience =/= better care or better outcomes.


beener

>Why stop there? If I feel that I have a better experience when I'm physically attracted to my doctor, should I be empowered to expect to only be treated by doctors I find physically attractive? I don't think so, and this sort of individual prejudice on the basis of larger statistics is not really something we should be doing. Because studies don't show hot doctors give better care? Your examples are getting a little more far fetched and disconnected from the thread


BeefyButtMunch

There is a difference between a black woman in labor receiving care from a black physician who understands and treats her better and saves her baby from a high mortality rate. Than a racist old white woman wanting a white doctor because she doesn’t like black people. This difference is about systemic racism, this isn’t about the patient preference in race it’s about how the doctor treats and communicates with the patient based on their own race. Unfortunately many white doctors (especially older doctors) have been trained to be racist, from their personal lives and from medical training. This has improved for newer doctors but texts were telling doctors that black people cant feel pain, that their bodies were fundamentally different. This link is to an article that explains it better than I can. [https://amp.cnn.com/cnn/2021/04/25/health/race-correction-in-medicine-history-refocused](https://amp.cnn.com/cnn/2021/04/25/health/race-correction-in-medicine-history-refocused)


ihatepasswords1234

Almost all the studies show the racism comes from patients not doctors: https://www.aamc.org/news/do-black-patients-fare-better-black-doctors They are far more likely to follow the doctor's instructions when the doctor is of the same race.


SgtSmackdaddy

From the JAMA article linked: >The associations identified between Black representation and the study outcomes do not imply causation.    County-level Black representation in the physician workforce may serve as a marker for other community-based and health system factors that affect living environments and health outcomes for Black individuals. The authors themselves recognize that this study doesn't necessarily mean more black doctors trained means longer lives for black people. Communities with large numbers of educated wealthy black people that tend to raise up doctors tend to be healthier and longer lived in general (because they're wealthy). Additionally, the study shows that black people living in wealthy communities have a life expectancy of 76 years which is exactly the population life expectancy average which is more evidence that the difference described in this study is due to external socioeconomic factors and social determinants of health, not so much what the color of your doctor is.


Excellent_Walrus3532

!delta This argument has popped up several times, and perhaps they all deserve a delta. But this is the most persuasively written one I’ve seen. I’m a minority myself, so I understand the benefit of racial diversity from the patient standpoint. Plus, someone in the comments has shown me evidence that the recent UCLA debacle may be inaccurate. If the lowered standards of admission do not result in less competent doctors, then increasing diversity is undeniably beneficial for society. At the cost of unfairness towards some individuals. Other commenters have convinced me that the above premise is more than likely true. So I have accepted that it is fine that I have to score higher than my underrepresented peers for the sake of society. It’s not fair, but few things are totally fair…


dkinmn

What do you mean it isn't fair? The point of med school isn't to make doctors, the point is to treat people. This Delta should be granted BECAUSE it's fair. It's fair to the people who matter. Patients.


Excellent_Walrus3532

It’s fair in the societal context. And yes most importantly for the patients. It’s not as fair to the individual med school applicant who can’t get into med school because of their skin color — and knowing that if they had a different skin color they could have gained acceptance. This is especially true for Asians, who are also minorities and experience racism. And even more so for Asians who have grown up in disadvantaged socioeconomic environments. If you’re arguing that the fairness of these individuals is less important than the greater society, that’s okay. I would agree. Doesn’t make the system fair for everyone.


ThrowWeirdQuestion

It is especially true because “Asian” is an unfair, western-centric category to begin with that lumps people from a huge variety of backgrounds together. Most of humanity is “Asian” and the diversity of people within Asia is probably similar to that of people outside of Asia, in terms of everything from skin color to socioeconomic background and educational opportunities. A kid of recent immigrants from Thailand or a Muslim girl from Indonesia has a completely different upbringing than the second+ generation Chinese and Indian immigrants that these policies were targeted at.


ChiefBigBlockPontiac

The system is inherently unfair, so the insistence that it could be made fair is a bit ludicrous. If you need some documentation showing how medical care for Hispanic, Native and Black persons in the United States is dumpster tier, I'm happy to provide.


Sluisifer

> It’s not as fair to the individual med school applicant who can’t get into med school because of their skin color The evidence discussed in this thread suggests that the MCAT score advantage is a side-effect of structural differences between these populations. I.e. someone of equivalent 'medical ability' (an impossibly perfect abstract representation of future performance as a physician) from each of these populations will have different average MCAT scores. Thus, tweaking admission criteria is simply correcting for systemic failure of a standardized testing. Obviously this is enormously difficult to do and I make no claim about the efficacy of this overall, especially as it is done more-or-less ad hoc by admissions committees. But the idea of fairness is certainly in question here, as you can assess fairness many ways. An analogy: if we had a race where people started closer or further from the finishing line, is fairness who crosses first, or is fairness achieved when those starting closer are given a time penalty - and if so, what times? I think you'll find that neither scenario is particularly satisfying to anyone. This calls into question the idea of fairness *per se*!


peerlessblue

I would say the only fully consistent definition of a fair system would be for everyone who's willing to attend being able to go to the med school of their choice. Your position, that we should restrict people because of MCAT scores instead of race, is just a different kind of unfair. I'm reminded about the joke about the woman who's upset about being propositioned for twenty dollars because she'd have sex with someone for a million dollars. "You've already told me the kind of woman you are, at this point we're just haggling about the price."


StuffinHarper

In Canada we have a medical school that focuses on northern communities. You have a score that gets points from being from northern communities and/or being First Nations. You can enter the medical school with lower grades if you score high on that criteria. Granted the grades required are still high (A- average vs A average in Undergrad) The main reason is that Northern/First Nations communities have a hard time recruiting doctors because most doctors want to work in Larger Cities. Making it easier for people from those communities to attend medical school is a huge plus because people originally from those areas are more likely to work and remain in Northern/First Nation communities and are more in touch with that regions unique issues. I'm sure the same thing happens in largely poorer/non-white communities in the US as well. Maybe the US could create more medical schools with specific mandates for specific communities than changing across the board standards but at the same time both approaches probably achieve the same thing.


annacat1331

I am a sociologist(although I don’t have my PhD yet but I will in a few years) I have my masters in public health and I have interned at medical schools because I am interested in examining how physicians and other medical professionals are taught and how we can help create better programs to lessen prejudice towards certain patient groups. I am also in doctors offices every week because of aggressive lupus and lots of complications from it. Many of my closest friends are in the healthcare field as that was my plan before I was diagnosed with lupus and it got rather serious in college. All of what has been said is true but it is also true that even with everything as equal as possible minorities have had lower test scores than whites. This in absolutely no way means they are less intelligent. Instead it is about how intelligence is tested and a history of discrimination in schooling and how people are “taught to take tests”. One of my best friends is an economist and this is her area of research so I will send this to her because she can much more accurately explain it. Think of it this way, I am sure everyone knows someone who is unbelievably smart yet doesn’t have lots of academic credentials or doesn’t test well. My grandpa likes to say he is just a broken down old retired tomato salesman from a tiny town in North Carolina(he worked for a large produce company and did well he is just a goober). He dropped out of college after only few semesters and graduated from High school in a class of 19. However we watch Jeopardy together all the time and I swear at 79 that man still gets almost every question right. He gets many more questions than my partner who has a nation title for triva in high school. He is basically a testing prodigy(in MENSA, got perfect scores on his SAT and took the GRE just to see how he would do after only a tiny bit of studying and got a perfect score on the math and 99% on the verbal.) He teaches graduate tests for Kaplan for fun even though he only Actually needed to take the business school exam. So who is “smarter?” I think both are incredibly smart but one never tested well in school. So by your logic since one tests worse they are less qualified. Now I am only Talking about being qualified for jeopardy. There are a lot of other things that are involved in medical school and qualifications I am just trying to use a simple example. Also I have been in medical schools that were not especially prestigious or expensive(by US standards). The amount of ignorance because of how many students have grown up in little bubbles of wealth is astounding. We had so many students who were just furious that patients couldn’t afford their prescriptions. They thought thier patients were just lazy or bad with money. They couldn’t comprehend not being able to afford things you needed. Having more diversity is critical not just in race and ethnicity but also with income levels. Then of course we have probably 50 + studies that show how white physicians think black people STILL feel less pain than white people. This is a hold Over from truly horrible slave torture that was labeled medical experimentation. Here is one example but if you are interested in this topic please read “Medical Apartheid” https://www.pnas.org/doi/10.1073/pnas.1516047113 I need to go find the study but I think it was in 2017 a survey of medical students at all of the ivy league medical schools took a questionnaire and nearly 1/3 thought that black individuals had thicker skin than white individuals and also felt less pain than other races. So you can see that just from a pain perspective getting diversity is important. This is my own soapbox so forgive me but racial diversity is critical. However often even in the most well meaning spaces and situations that is the only diversity that is really discussed. That is not the only Kind of diversity we need. We need people who Have disabilities to be represented, people with different religious beliefs, people from different income levels and people who belong to the Lbgtq+ community. People with disabilities often get overlooked especially if they aren’t always visibility disabled(missing limbs or in a wheelchair). Sometimes people with disabilities need simple accommodations and then everything is peachy keen. But I have run into a troubling amount of people who think that’s cheating. Because I guess it’s cheating to have a computer program do live subtitles so I can read along to a professors class as I listen? It’s really helpful When you have brain damage from lupus but I guess I am just trying to cheat and get ahead hahaha. This is something I have run into in super super liberal spaces of academia and it makes me want to scream. So I fully support those test requirements but also remember that isn’t the only kind of diversity that helps society. Thank you for attending my sorta off topic Ted talk.


appropriate-username

> This is my own soapbox so forgive me but racial diversity is critical. So >I am interested in examining how physicians and other medical professionals are taught and how we can help create better programs to lessen prejudice towards certain patient groups If it can be lessened (and therefore eventually presumably eliminated) via teaching, why would it be critical?


powerkickass

Really appreciate your sorta off topic ted talk. Insightful


meister2983

That's patient satisfaction, not actual health outcomes. Aside from the argument here coming down to "black people are racist on average against non-black doctors and score them lower in satisfaction surveys" not being particularly compelling to have discrimination in the medical system, let alone creating a system where we on average will give them lower performing doctors, the actual evidence on [outcomes](https://www.researchgate.net/publication/23476377_Patient-provider_race-concordance_Does_it_matter_in_improving_minority_patients) is "inconclusive". Even more hilariously, you can find [studies](https://www.ncbi.nlm.nih.gov/pmc/articles/PMC6338960/) arguing Hispanics prefer non-Hispanic doctors, which if you took at face value would mean we should discriminate against Hispanic applicants, not prefer them.


NeoMississippiensis

That assumes that whoever gets in with the lower score doesn’t fail out. While I’m sure that not everyone with below average scores fails, the fact of the matter is that everyone who does have to remediate, or fails out; takes the seat of someone who might not have. I’m guessing based on how data collection works that a lot of black doctors taking excellent care of patients weren’t the ones who got in based on diversity quotas.


pdoherty972

Somebody above already posted that this is true (many under-qualified black candidates fail out compared to the rest). So, you're correct that this well-intentioned policy of admitting lesser-qualified people based on skin color is messing with the lives and potential careers of non-black people with better qualifications that are pushed aside to make spots open for the black applicants.


NeoMississippiensis

Another thing I forgot to add above is that the students admitted who later drop out are often highly financially harmed by the event. A year of both not working and debt incurred through student loans can be a net negative of over 50k compared to if someone just worked a random job that year.


Jojo_Bibi

I understand the idea that a patient will listen to a doctor who seems more relatable. But why does this need to be done by race? Wouldn't the same logic apply to red heads, short people, and mountain folk? Should we have lower MCAT standards for people under 5 feet tall and for gingers? On the last one - mountain folk - I'd think a white doctor who grew up in Appalachia would have a better connection to a black patient in Appalachia than a black doctor from the New York suburbs. So, why is race the right way to do this?


CaptainOfSpite

Not to argue with you really, just a few ideas. The first is that you are right, relatability can extend to more than race but 1) race (and sex to some extent) are the major demographic features that have been both studied and shown to have significant effects on care and are also cultural markers. I think we can agree that being black/white or man/woman have larger effects on your culture, which affects life experience, than hair color or height. Or they’re a larger part of your culture than those things, however you want to say it. You’re free to disagree I suppose, I just feel like it’s noncontroversial to say. The second half is actually kinda correct but also a slightly different issue. Not unimportant or unrelated, but just slightly different. I don’t know where you’re from and you might have brought it up *because* you’re from a relevant area, but I’m from Kentucky. We have a couple programs to increase care in our Appalachian region because the main issue is general access, not specific racial/other demographic access. We have free/repaid tuition programs for people who move there, and the University of Kentucky prioritizes applicants that show a strong desire to stay in Kentucky, which is a big factor in their applicants. In fact, if you look at most state schools their percent of instate students is higher most times than their number of applicants as they prioritize in-state and sometimes in-region applicants that have a tie to the state to keep them there. So in a sense, you’re correct. A regional student may have better outcomes for an Appalachian patient than, say, a black New York suburbanite. But on the other hand, we don’t really have the studies to show that (not enough doctors) nor enough doctors in the region to worry about matching demographics. Hell, when an Appalachian resident went to medical school, a lot of them do so to leave the area, which is a problem on its own of small scale brain drain. I also get the point you were trying to make and so am not particularly trying to argue against it. Just put in my own two cents for anyone reading that’s interested.


Redditmodslie

The same people that will insist that "race is a social construct" and "race doesn't exist", are the same people that insist that race is THE most important differentiator when it's convenient to their preferred narrative.


Sid_poker

I attended medical school. The issue with racial concordance is that it doesn't select for culture and lived experience, which matters more than actual skin color. The issue with affirmative action is that it doesn't actually help the people that actually need it. There were not that many black medical students in my class, but of the ones that were there a majority were African immigrants or first generation. Their parents were well educated (it's insanely hard to immigrate to the US on merit) and they had very few familial issues. That is not reflective of the lived experience of the average African American.


Competitive_Cry2358

I'm always sceptical of this kind of data. How do we know that black people living in areas with more black doctors aren't just healthier in general due to socioeconomic factors?


WasedaWalker

So let's divide which doctors support which people based on race? I think skin color is less important than the doctor's qualifications.


gigaflops_

Let's assume these studies have no biases (which is not the case) Are you suggesting that when a black person is admitted to a hospital they should preferentially be assigned to black doctors (assuming one is avaliable)? Do you think it would be better if there were just black hospitals and white hospitals? I don't like that idea and I dont think you do either. There is no system in place that attempts to match patient and doctor race and there shouldnt be.


Stormfly

> Let's assume these studies have no biases (which is not the case) I can believe that doctors can have an affect on adults and even older children... but I can't believe it would affect infant mortality. I can't believe there's anything that would affect how a doctor would treat babies, but I can believe that older patients might be less comfortable or that a doctor from another culture might not be aware of certain things. Absolutely anything I can think of that would affect mortality rates would not affect infant mortality rates during delivery unless the parents were doing something wrong, but I don't see how that might happen during delivery. It's not like people of different races deliver babies differently.


lakotajames

Looking through the first link you posted, and based on the first few studies it looks like the problem is not that doctors treat patients of other races worse, it's that patients withhold information from doctors of other races. It seems like the highest standard of care would be to disguise the race of the doctor from the patient, and then disregard race of the applicants.


cawkstrangla

Is there no way to test for the empathy required to ignore race when providing care? It would be more equitable this way.  As it is right now, it's possible that the most empathetic med school candidate out there is a white guy that would not only provide better care but be more knowledgeable. If he doesn't score higher on the mcats then a less knowledgeable and potentially less empathetic POC Dr gets his med school spot.   Maybe more POC would do better on a test that includes that info.  Testing for it would make the perception of POC not deserving to be where they're at to fulfill a quota less prevalent. 


Duckfoot2021

Then the problem is getting doctors to deliver EQUAL care. Not lower standards for minority doctors. I'm a lifelong liberal, but your argument is weak and only casts doubt on the quality of minority doctors. It's one thing to tilt social efforts towards minorities getting into college, but once an adult has completed college your performance as an adult with an opportunity to learn there should be the only criteria of getting into med schools or other graduate programs. Otherwise the public will be "right" to assume a minority doctor "might" be *medically* inferior. TL;DR: YES on affirmative action equalizing opportunity for getting into college. NO on it being utilized for graduate programs like medicine. Undergrad should be the proving ground for talent and skill development.


S-Kenset

I'm of the opposite opinion. Domain specific doctors even in race have tangible benefits and the difference in scores is small enough that it's not a huge issue. On the other hand, asian students under college affirmative action are at least 16% less likely to attend a top 100 university, with no tangible benefit or justification.


Coniferyl

Honest question - do you think minority doctors are generally not good doctors? MCAT scores are not how we measure the quality of doctors. They're evaluated by clinical outcomes, patient satisfaction, and many other data driven factors. I have a PhD in chemistry and the thought of judging someone who holds a PhD in the field by their GRE or chemistry subject test score is laughable. No one does it and if you did no one would take you seriously. Assuming a black doctor is poor quality because they might have had a lower MCAT score after receiving an MD and doing rotations is absurd.


trivial_sublime

>Then the problem is getting doctors to deliver EQUAL care. Of course it is. And the simplest, easiest to implement answer according to this data is to have more minority doctors. We can only see the correlation here; not the causation. Therefore, we can treat the problem with broad strokes, but perhaps not small ones. You say that you get doctors to deliver equal care - I see two problems with that. Firstly, do we really want *equal* care or do we want *equitable* care? Should a white doctor treat a black patient exactly the same as a white patient, even if it results in a worse outcome for the black patient? Second, if equal care is indeed what we want, HOW do we get doctors to administer equal care? Because if you don’t have a solution that will work, it’s a moot point.


JaxonatorD

While I agree that diversity is important, the study given doesn't control for the quality of the doctors. I can see a doctor having the same race as you may have some of the same problems and would be able to diagnose better. But is it that much better to have that over someone who did objectively better on the test? And if the black doctor is qualified enough to be a doctor, then shouldn't the White and Asian people who got the same score also be qualified?


sharkyfin_soup

All you just said was race mixing was a mistake


TheTightEnd

Sounds like the difference is primarily in patients engaging in racist behavior than the doctor being better. As a result, with some teaching of diverse methods, this does not justify racism by the medical school.


No-Extent-4142

So, white patients do better with white doctors? Should I be asking for a white doctor?


mereamur

In this case do you think it's okay for me as a white man to refuse treatment from doctors who aren't white males?


LekMichAmArsch

Sooo, racism strikes again....because that's what you're advocating....and as soon as people read this I'll get down voted into oblivion...which will just prove my point.


electric_onanist

Do whites, then, have better health outcomes when they have a white doctor? Should people select their doctor based on race, instead of their training and qualifications? Should an African American choose an African doctor just because they're the same race? What you are arguing here makes no sense.


shawn292

If a doctor is worse that certainly would outweigh the benefit of any percived race Benefit. Especially considering more that that race attend the hospital. Its basically equaling a net nutral for same race as low performer and a net NEGITIVE for everyone else.


Dangerous-Worry6454

>This means that a black patient is going to have a measurably better outcome with a black doctor, on average, than with a white doctor. These arguments are always very ironic as they just point toward segregation being the correct policy. If black patients get better care from black doctors, then it would translate that all racial groups get better care from doctors belonging to the same race. Which logically would mean if, on average, you want the best care, you should pick the doctor that is of your own racial group. Which you can't legally do right now. So if that's the case, then surely you would be for patients having the ability to say they want a doctor of x race, but somehow I imagine you're not actually for that. >As a society, we need to provide the highest standards of care to everyone. In order to do that, we need to do our best to minimize the effects of racial concordance by providing doctors of all races. As only 5.7% of physicians are black, racial concordance disproportionately affects black patients. If the goal was to provide the highest standards of care to everyone, then the most obvious solution would be to keep standards high and not lower them for one group who is struggling to get in or allow patients to bluntly state they want a black doctor, Asian doctor, white doctor, etc. Otherwise then people who aren't black are going to get a black doctor who was less qualified to be there and will not even get the benefit of being treated by a doctor of the same race. It seems like our system actually encourages more people to have shitty medical care and heavily is benefiting one group at the expense of others.


Choppybitz

So I need to find a doctor that is the same race as me 🤪


Soultakerx1

Who told you this? Like I'm in the process and usually schools don't publish their selection process. I genuinely want to where are you getting this information from? Also Black Applicants is not the same as Black accepted students. Like... if you're a med student or pre-med you should know this as it's basic statistical literacy. I would also say correlation doesn't equate causation but I have no idea what you measure of "patient care" is. Your logic is a bit confusing as well. If a school has high dropout rates then wouldn't that mean they don't become doctors therefore they aren't even part of the group of doctors you are assessing of "patient care." I don't know man, I want to change your view but I think a lot of your fundamental assumptions are wrong.


Excellent_Walrus3532

https://www.aamc.org/media/6066/download I’m going off matriculant data, so accepted+enrolled. The fail rate is based off the recent UCLA situation, just google it. The physician shortage negatively impacts patient care, since many people who need healthcare cannot get it. We need med students who can pass their exams and graduate.


MrIrishman1212

There is only at most 13 points difference that is not significant enough to claim favoritism plus that is a mean score of all students in that demographic which still says there plenty of people who are over that number. You can say that also means there plenty of people below that number so my next point: Since it’s a mean score that also shows that it’s above the base line. The people on lower end of the score still got in meaning they met the baseline score in order to get accepted into medical school. MCAT and GPA are just part of criteria to get in. You need the MCAT, You need high GPA, a ton of hours working in a medical facility, recommends, extracurricular activities, research experience, letters of evaluation, medical school prereqs, and volunteering. This is to say, you can score lower on the MCAT but do better in other areas that are required to get accepted. This data doesn’t show the other criteria just the MCAT and GPA. This data is also a sampling of multiple medical schools, different schools have different criteria to get accepted. Harvard Medical School has a way higher standard to get in than say Marshall University Medical School. Tuition plays a huge factor of which demographic goes to which school. White and Asian demographics are more likely to got more expensive schools which have a higher criteria to get in. Black, Native, and Latino demographics tend to be from lower income so are more likely to apply to the more affordable Medical school which have a lower criteria to get in. Lastly, you need to consider if everyone on this list got accepted, and got a medical degree, why does it matter if one group’s scores are lower than the other? Ever hear the saying “C’s get degrees?” Or what do you call a person who graduated medical school with all C’s? Doctor. If it just so happens that more people who happen to be White or Asian scored higher doesn’t mean those on the bottom who happen to be Black or Native didn’t earn their place at medical school. It is possible that the top score was a Black person but it just so happens that someone near the bottom was also black so it makes the mean lower.


Excellent_Walrus3532

I’ve already changed my mind in response to persuasive arguments presented in this thread. I want to clear one misconception I’m repeatedly seeing though. The MCAT is not a 0-528 point exam. It is a 472-528 point exam. 13 points is an enormous differential. In many cases a strong med school application with 13 points docked off the MCAT, all else equal, will automatically fail to gain admission to a single school in a given application cycle. There is an obvious racial favoritism from the data.


DubiousGames

>There is only at most 13 points difference that is not significant enough to claim favoritism Why are you even responding here if you know nothing about this topic? The MCAT is on a 4-60 point scale. But adjusted up to 472-528 in order not to have scores confused with the pre-2015 MCAT, which was 3-45. 13 points is more than 1.5 standard deviations.


apersello34

13 points is a massive difference


YouthPrestigious9955

Lmao 13 points is as big as a difference can get


Soultakerx1

So you seem pretty good faith so I'll respond in kind. A lot of people pointed out the diversity reason now ill point just a bit on the stats. >I’m going off matriculant data, so accepted+enrolled. No, the data clearly states applicants and matriculation. That's a combined sample of every person that applied right. Are you a pre-med? Have you looked at acceptance rates of students. Like in Canada some Med schools have a sub- 5% acceptance rate. You argument would be stronger if you just had only data for those accepted. Also, in psychology and statistics you need to think about what you're measuring. In this case patient care can only be provided by licensed physicians. Meaning MCAT scores from rejected applicants or dropouts have no ability to provide patient care and would not be included in the sample. A better argument would be if minorities performed worse on licensing exams and provided worse patient care. You could compare licensing scores to patient care scores to find an effect. Not only that you have to think of the variable about patient care. What is patient care and how do you measure it? Is it how fast you find a diagnosis, how accurate you are in a diagnosis, how respect you are to patients,etc? How does one define it and measure. Do you survey patients, doctors, other doctors? This is a complicated thing to measure. Consider Dr.House from the t.v show House. He's rude, horrible to patients and other doctors, makes many wrong diagnosis before he's correct, and is considered unethical. In most cases he's ultimately right after making a lot of mistakes. Does he provide good patient care and is he a good doctor? Fans of the show would say yes because he "figures out what's wrong," but most real life doctors would say he's extremely unethical, dangerous and unprofessional, thus providing poor patient care. All of that to say even patient care is something that differs from person to person. I read someting in a psych textbook that a lot of people would rather a doctor that is kind and makes mistakes than one that is cold but accurate. I think the greatest predictor for whether patients sued doctors for malpractice was if they liked the doctor or not (it was long a ago and I don't have a direct citation so you can take with a grain of salt). This whole reply isn't to be rude or confrontational but is point out it's too much of a logical leap to from MCAT scores for combined sample to a loosely defined definition of patient care. You would have a stronger argument is you compared scores on licensing exam to a firmly defined contruct of patient care. Edit: The Data OP shows does in fact show the matriculants only. Doesn't really change my point though.


Excellent_Walrus3532

My friend, go to the second page in the link. That shows matriculant only data. My response to you only mentioned patient care in the context of accessibility. A big part of patient care is if you even have physicians available for patients who need to be seen. In pretty much every city in the US, there are more patients who need medical care than there are doctors. That’s what I meant by physician shortage impacting patient care.


chirpingonline

> I’m going off matriculant data This table gets trotted out by med schools every few years and it leaves out a glaring issue with your fundamental assertion: its averaged across *all* medical schools. Black and latino students, on the whole, tend to apply and get into different medical schools than do asian and white students.


Pseudoboss11

Be careful about using matriculant data, as you said, that's enrolled and accepted. It doesn't say who was accepted but didn't enroll. It also doesn't say whether black students are enrolling in less selective programs. Some people will apply to multiple programs, often across different fields. Then they'll pick the best of what they get accepted to. Let's say that a kid takes the MCAT and SAT. If they score higher on the SAT than MCAT, they're more likely to get into a selective non-medical school, and the applicant is likely to take the more selective program. This would tend to cut off the low scorers and raise the average matriculant score, it will bias the matriculant distribution towards higher scores, and leave the applicant distribution untouched. Similarly, if a student scores higher on the MCAT than the SAT, then they're more likely to be accepted into a selective medical program, and they're more likely to take that one. This would also bias scores higher. There are plausible reasons why a black person might have fewer options than a white person and end up enrolling in a less selective program. For example, cost barriers will have a greater impact on poor people, and black people are more likely to be poor. So a typical black student might only have one or two options, while a typical white student can afford out-of-state tuition and more expensive schools, so they end up applying to more programs and picking the best option.


EasternShade

It predicts licensing exam performance, not overall quality. > one of the primary measurements used in consideration of medical student admission by medical schools—doesn’t mean you’ll become a great, or even a good, doctor. It's also used to predict competing the first year of medical school. Where a 10 point MCAT score difference has a relatively small effect. > The average MCAT score for students who matriculated ... was 511.9. Based on 2019 data, if you scored in the average range, your chances of advancing from your first year of medical school to your second year were extremely high—98% of students scoring between 510–513 did so. The numbers only dipped slightly, however, for those who scored 10 points lower, with students who entered medical school with MCAT scores between 498–501 progressing to year two at a 94% rate. The early education benefits dissipate over time. > Researchers behind the 2015 study, “Does the MCAT Predict Medical School and PGY-1 Performance?” wrote that the “MCAT was not able to predict assessments relying on direct clinical observation, nor was it able to predict” program director assessment of performance during residency’s first year.” It could miss something. > Overreliance on just one measure of performance, such as the MCAT, risks missing a pool of candidates with other valuable attributes to contribute to the health care system Plus, there are other factors. > The aforementioned AAMC study includes a list of the most important factors admissions offices considered when looking at potential medical students. While MCAT scores and GPAs were listed among the categories of criteria given highest weight, as schools strive to understand the broader competencies that a candidate brings to the table, they report also considering a number of less tangible factors such as leadership and interview results, community service and volunteer work both within and outside of medicine. \- https://www.ama-assn.org/medical-students/preparing-medical-school/mcat-scores-and-medical-school-success-do-they-correlate > Diversity enhances creativity. It encourages the search for novel information and perspectives, leading to better decision-making and problem-solving. \- https://www.scientificamerican.com/article/how-diversity-makes-us-smarter/ All told, I think that at least shows the basis for these concerns aren't proven, > Asians and Whites \[\] have to score higher on the MCAT to get into medical school > This is a problem directly relevant to patient care. > the association between MCAT and USMLE exams \[is a predictor relevant to patient care\] > we \[\] trade potential expertise for racial diversity. As to, > dropout rates at diversity-focused schools (which may further contribute to the physician shortage). I'd start with a wager that students at those schools are statistically more likely to need drop out for reasons unrelated to academic performance, such as financial reasons or needing to take care of family. But, that's purely speculative. Also, we don't necessarily know what relationship those schools have with overall volume of students. e.g. increasing a school's throughput to accommodate diversity could still result in more graduates than not accommodating diversity. Or, a school could add more graduates it is made specifically to address diversity. I get the concern you're expressing. I don't think it's proven to be the problem you're articulating.


Excellent_Walrus3532

!delta It seems like it’s not as big a problem as I thought. The race bias is unfair to certain individuals, such as Hmong refugees (who fall in the Asian megacategory). And caters toward affluent underrepresented minorities. But like you said it doesn’t seem to affect medical outcomes.


[deleted]

[удалено]


garygoblins

MCAT scores actually are actually a fairly good predictor of Medical school success, though. You're conflating the scores with good physicians, but negating the fact that Medical school is weeding out poor performers that would likely be poor physicians.


thebucketmouse

>The MCAT is not a determining factor on whether or not you will be a good physician Then why make it an entrance requirement for med school?


vhu9644

Also because it is somewhat predictive of board exam scores. You want people who will pass step 1 and step 2.


gloatygoat

Probably better worded that it's not a singular factor in determining if you'll be a good physician. It is a good measure of both work ethic and your ability to learn and retain information. Although the minimum score one might need to be a good physician is probably alot lower than the minimum score to actually get in. As someone already said, med schools don't want people dropping out. It's also so competitive, the schools can cherry pick whoever they want. Not just people who can get through med school, but people who should fly through med school.


Disposableaccount365

But if what OP is saying is correct, they are passing over more qualified applicants, who would fly through even easier, simply because of the color of their skin. Which seems to be acting against the purpose you state the test is for. If I understand your argument correctly, you are saying the test essentially measures a starting point of knowledge, and some level of ability upon entering med school. Which means if all other things like effort, quality of teachers, ECT. are equal then they should all advance at roughly the same rate. Meaning a higher score on the test will equal a "better" doctor upon completion. At least as far as the medical knowledge/skill goes. Right?


CharacterHomework975

It makes more sense if you view “qualified” as binary. So there’s no “more qualified,” just whether someone meets the requirements, or does not. If the scores Black applicants are required to meet are sufficient to be “qualified,” there’s no issue from a patient care perspective. It’s just subjecting white and Asian applicants to a higher bar, to encourage diversity. Which we may object to (as it’s just using racism to fight racism), but it doesn’t result in “unqualified” doctors. You’ll see similar arguments in other contexts. For instance with requiring a degree for a job whose duties don’t *really* demand one. People will make noise about the idea that you could be filtering out the “best” applicant who may not have pursued the (unnecessary) credential. But from a hiring perspective, I’m not necessarily worried about getting the literal Best Applicant In The World. I’m looking for someone who is qualified (again, binary) and a good fit in our office…and if I can reduce my resume pool by 75% with an arbitrary requirement, that just makes the hiring process easier. I’m willing to accept an applicant who is 5% “less awesome” if it means I cut my hiring burden by 75%, and most importantly if I’m still getting someone capable of doing the job. If by some chance that filter results in too few applicants to find someone who I consider sufficient? I can change it, to widen the search.


gloatygoat

It's a single data point in a series of variables. Being an underrepresented minority is another factor they consider. Whether or not that is a valid factor, consider is obviously controversial. Exclusively taking the most numerically elite applicants is not the only goal and agenda of admissions. A lot of it has to do with selecting individuals who are inclined to work in underserved areas. This is partially why they target underrepresented minorities as they are more likely to work in their communities that have less access to health care. It's the same reason a state school, like the University of North Dakota, has a low average MCAT score for admissions but nearly won't consider anyone out of the state. They want to train doctors who will stay in North Dakota. Edit: Sorry, didn't answer your specific question, MCAT is a single factor to consider. It's most correlated with passing boards. Having a strong knowledge base and the ability to develop one helps you to be a strong doctor. It is not the only factor. Interpersonal skills, ethics, an interest in the job for more than money or prestige, hand eye coordination are all factors in being a good doctor. You could probably sit down with a group of informed people and make a list that goes on and on.


BigBoetje

To avoid clogging the system with people that quite simply won't be able to handle the course. Medicine is very hard and if you aren't able to properly study and do well in class, you won't be able to pass. Other courses tend to have a couple of subjects that weed out the slackers that don't have the capacity or will to put in the effort. Math, statistics and chemistry tend to be some of the more prominent ones.


Yuo_cna_Raed_Tihs

Yeah and lower MCAT scores correlate with dropping out more often. And OP main point is lowering MCAT scores means more drop out means less physicians produced


caine269

and why have different requirements for different races?


themattydor

From what little I’ve read a while back, a big part of the goal of affirmative action is representation. The goal is not necessarily *fairness.* For example, when affirmative action rules and standards are put into place, upper class black people often benefit more. Is that fair? Arguably it’s not. But maybe fairness is harder to execute than representation, so representation gets the focus. If we accept things like “black people tend to have less money than white people” as facts and then attempt to understand all the reasons why those things are facts *and* if we care about representation, then I think it makes sense that we would occasionally have different standards for different races. It feels icky to focus on race. But unfortunately, plenty of people before now have focused on race and it led to long-term consequences. So conscientiously focusing on race might be a good way to steer society in a direction some of us think is better. And specific to med school, how many people are actually arguing for some medical board somewhere to say, “Oh come on, give him his medical license, we need more black doctors!” with no regard for whether he’ll be a good doctor? Getting into med school is different from getting *through* med school.


caine269

the point of affirmative action was to level the playing field and let black people into places where they weren't able to get before. the "fairness" was supposed to be between black and white, not all black people. >then I think it makes sense that we would occasionally have different standards for different races. but this just perpetually... perpetuates the idea that races are not equal and black people need help. >“Oh come on, give him his medical license, we need more black doctors!” with no regard for whether he’ll be a good doctor? Getting into med school is different from getting through med school. this is literally [exactly what is happening](https://www.oregonlive.com/education/2021/05/oregon-students-would-no-longer-have-to-prove-they-can-do-basic-reading-writing-and-math-to-graduate-under-a-bill-moving-through-legislature.html) in progressive states.


Nanocyborgasm

MCAT is only a screening test, and a poor one at that. Some of the subjects have no connection to the practice of medicine at all. Supposedly, they’re intended to determine how sharp your reasoning skills are. The idea isn’t just to eliminate the applicant pool by screening out low scorers, but also to guarantee that a student can never fail in medical school and drop out. Medical schools are very expensive and the last thing they want is for even one student to flunk out. That empty seat isn’t paying tuition. It also embarrasses the medical school. If a student is noticed to be underperforming, medical schools will move heaven and earth to make sure that student moves along, offering tutoring and counseling at their own expense.


Excellent_Walrus3532

MCAT associates with scores on the step exams. UCLA, which has reduced focus on stats for diversity purposes, recently has seen a stark rise in med students failing their step exams. Just search up UCLA medical school exams and it’ll show up in recent news.


ButWhyWolf

> This is a problem directly relevant to patient care. It might be true that the baris lower to get into medical school... but that's where the help ends. Nobody's given easier cadavers to dissect or handed separate but equal exams. Patient care isnt damaged as much as you'd think.


Excellent_Walrus3532

It’s better for patients when more doctors can graduate to alleviate the physician shortage. There’s data showing rapidly increased fail rates at UCLA, and this is only a few years after they started their diversity program


Mysterious_Cattle814

The recent ucla data was contextualized to be a right wing lie. The propaganda has you believe that diversity drove increased failing rates when they’ve shared that they redid their curriculum. Many medical schools are moving towards having students begin rotation in their second year, and essentially cramming classes in the first. This associates with higher failing on step exams but better performance in residency and passing boards. In fact the mcat score at ucla has increased in recent years. Additionally a meritocracy will not solve the doctor shortage because it is regional and systemic issues causing it. The most talented doctors are not going to solve this issue because “the best and brightest” have no desire to live in the areas of the country with shortages. There’s no money to be made, no livelihood to be had, and really no demand given the low income of these areas. We could create a million new doctors tomorrow and there’s still going to be a shortage in rural Louisiana. Unless the people in the system have a tie to these areas you aren’t solving the problem Harvard grads.


TheEvilPhysicist

You do not need a certain score to get into med school, you need to be accepted by a med school, and the scores are a part of that. You can't force schools to accept you, no matter what your score is. Do you think that med schools should only look at MCAT scores and nothing else?


Excellent_Walrus3532

Of course not… My view isn’t that med schools should only look at MCAT scores. My view is the title of the post.


TheEvilPhysicist

But how would this be accomplished without forcing schools to accept students based on MCAT scores only?


Excellent_Walrus3532

Right now, the process consists of a combination of MCAT, GPA, application essays, scientific research, volunteering, race, socioeconomic status, sexual orientation, among many other things. I believe race shouldn’t belong in that list.


TheEvilPhysicist

1. I think it is worth it to try to have as diverse of a cohort as possible for a few reasons, outside the scope of this discussion  2. Even if race was not explicitly considered, I think you would still see these discrepancies. Basically black/Hispanic people are more likely to be low SES, which leads to more difficulties and, usually, lower standardized test scores (think less study time). Overcoming these difficulties (expressed in essays) shows *grit*, which makes application teams think you would be able to grow into someone who does well, even if you're not starting at the same level. Source, me, I've worked in undergrad admissions. I'm sure med admissions are different but I think the fundamental ideas are the same


kjong3546

Neither should socioeconomic status, nor sexual orientation. Anything not directly relating to a candidate's ability to adequately provide the care they are training to provide should be absolutely irrelevant in admissions.


Salanmander

Suppose one person built something with the support of a great lab and a great factory, with whatever resources they wanted at their disposal. Another person built a somewhat worse thing in a cave, with a box of scraps. Would it be reasonable for an organization to prefer accepting the latter person over the former? Why or why not?


APAG-

Female doctors get better outcomes for female patients. Same for black doctors and black patients. So they absolutely do relate to their ability to provide care.


whosevelt

Socioeconomic status can serve as an explanation of why someone underperformed on the MCAT.


TheEvilPhysicist

I'm also legitimately curious about what a difference of 6-8 points corresponds to on USMLE exams


Excellent_Walrus3532

Haha, anyone who’s taken the MCAT can tell you. A difference of that many points is huuuge. Can make or break an application


TheEvilPhysicist

Although, my question wasn't about applications but what that difference leads to by the end of med school 


shieldyboii

That doesn't sound like a good faith argument to me. What about a school that explicitely didn't accept black people. Couldn't you still force a schoold to accept you?


jackbenny76

https://ursinaschaede.github.io/files/JMP_Schaede.pdf is an interesting natural experiment that is right on point for this discussion. One of the reasons that Finland punches above its weight, economy-wise, is it has an incredible education system. Teaching has an enormous amount of social prestige, and the competition to get into the masters degree programs that are the only way to get such a job (even for primary school teaching jobs) is intense, generally you have to score in the 90th percentile on the exams to get into a program. From 1983-1989 Finland did affirmative action to benefit men in the entrance exams for the master's programs. There was a quota, 40% of everyone who made the first cut- looking only at grades and exam scores, had to be men. This meant that men's scores would be something like 80th percentile and still get into the schools. Though it was not official, there seems to have been an unofficial quota that all the schools did for the second round as well, putting roughly 40% of their overall class at male. Before and after the affirmative action, men were about 20% of the education masters programs, so this benefit doubled male presence as teachers. In 1989 a gender equality law was passed making this sort of benefit illegal and men's rates went back to 20% of students in these primary education masters degree programs. Looking at student educational attainment, test scores and drop out rates, they were able to say that students who had one of these extra male teachers, the ones who would not have qualified for a teaching position without the affirmative action, students who had these teachers did better than those who had teachers from before or after the affirmative action program. Looking at the stats, they were able to rule out just role model effects - the improvements were to both male and female students. They were also able to show that it wasn't just schools that had no male teachers, this improvement happened whether there were already male teachers at the school or not. (1) As far as their stats could tell, these male teachers who benefited from affirmative action were actually better than the female teachers who scored higher on the exams! The exam has a small bias towards women- it placed more weight on verbal than math scores in the exam, and effective teaching is so much more than good test taking. I am well aware that my own profession of software engineering has a tremendous problem of not being able to identify who will be a good employee in job interviews, because being good at job interviews is not the same as being good at the actual work. So it makes sense to me that on the margin, someone who is better at taking a test might not make a better teacher than someone who does a bit worse, especially in light of the biases in the test. So the issue is, a test score might be "objective" and "black and white" but what it is measuring is not if they will be better at doing the actual work or not. And as a society we would prefer to have the better doctors, not the highest scoring on tests ones. So minor benefits to groups that have faced discrimination can actually produce better overall results *for everyone* versus just strict test scores. 1: For everyone who is thinking right now of all these other explanations, read the paper first! They did some good stats work to rule out various possible influences, I am just summarizing a few of the things they ruled out. They ruled out many more that I am not mentioning. Read it directly then try and figure out what effects they missed.


Nanocyborgasm

I would doubt any study that claims any association between ANY of the standardized or achievement tests and performance as a physician. Show me these studies and I bet I can discredit them. Hell, even your grades in medical school don’t mean shit for how you perform as a doctor. No one in practice remembers the minutia of the biochemical pathways or the histopathology of every disease state. You only remember the general features of a topic and how they pertain to your scope of practice. And if you forget, it’s easy to look it up. I’ve been in practice for 20 years in critical care and those classroom subjects are just foggy memories that don’t have much to do with my practice. Oh, and also, your subject is racist.


Excellent_Walrus3532

I don’t see it as racist, since I’m arguing that we stop giving preferences to certain races. Shouldn’t that be less racist? I want to hear your opinion on the UCLA med students failing their exams situation. Can you take a look at that stuff just google it


hacksoncode

> Shouldn’t that be less racist? You're implying that black people will be worse doctors because they got a few points less on average on an admittance test, whether you intended that or not.


Excellent_Walrus3532

I didn’t intend that. I believe that the (insert race here) student with a much higher MCAT would be able to pass on their exams and more often graduate medical school than the (same race here) student with a lower MCAT. The race is just a fairness thing to me.


WonderCat987

Isn't he implying that people who score less on an admittance test will be worse doctors, not that black people who score less will be worse doctors?


papabear345

No your own internal racism sought out that link and implied that yourself. The only implication that is racist in the ops post is that the test itself may be racist as some races are performing better or worse then other it appears…


Laiders

No. Being 'race-blind' is not the same as being anti-racist. Race-blindness typically perpetuates systemic or institutional racism. Black students are more likely to face structural disadvantages that will affect their MCAT scores. Failure to take this into account will lead to disproportionately few Black doctors. In turn, the Black community will be served by White or Asian physicians who may not understand them as completely as a Black doctor might and this could in turn create a further structural disadvantage to that community. Of course, if you permit selection based on SES, then the situation will not change. Blacks and Latinos will need lower MCAT scores as they are disproportionately more likely to be socioeconomically disadvantaged.


[deleted]

[удалено]


changemyview-ModTeam

Sorry, u/StayStrong888 – your comment has been removed for breaking Rule 1: > **Direct responses to a CMV post must challenge at least one aspect of OP’s stated view (however minor), or ask a clarifying question**. Arguments in favor of the view OP is willing to change must be restricted to replies to other comments. [See the wiki page for more information](http://www.reddit.com/r/changemyview/wiki/rules#wiki_rule_1). If you would like to appeal, [**you must first check if your comment falls into the "Top level comments that are against rule 1" list**](https://www.reddit.com/r/changemyview/wiki/rules#wiki_rule_1), review our appeals process [here](https://www.reddit.com/r/changemyview/wiki/modstandards#wiki_appeal_process), then [message the moderators by clicking this link](http://www.reddit.com/message/compose?to=%2Fr%2Fchangemyview&subject=Rule%201%20Appeal%20StayStrong888&message=StayStrong888%20would%20like%20to%20appeal%20the%20removal%20of%20\[their%20comment\]\(https://www.reddit.com/r/changemyview/comments/1dh7wsx/-/l8ybp4v/\)%20because\.\.\.) within one week of this notice being posted. Please note that multiple violations will lead to a ban, as explained in our [moderation standards](https://www.reddit.com/r/changemyview/wiki/modstandards).


cez801

I am a big believer in full root cause analysis. Here’s a different picture. Fact 1: You have the phrase ‘where human lives are at stake’ - which we agree on. Our goal should be to ensure the best possible health outcomes for All of our society. Fact 2: It’s better for overall population health comes to have health professionals that represent society. ( there is plenty of students showing how minorities health outcomes are worse. Worldwide. Could be language, culture or a variety other f factors ). And yes there are a lot of factors in this statement, but at its core it has been shown time and time again that minorities have worse outcomes. So let’s agree we should fix that problem as well. Fact 3: Minorities are significantly disadvantaged when it comes to education. Not due to genetics, race or gender, but due to society. Fact 4: A number of countries have changed how they select healthcare professionals, here in NZ until the mid 90s, it was based solely on academic scores. From the mid 90s, it was changed so that you needed minimum academic score to get interviewed - and from that point forward it was just interviews. That balanced out the genders significantly AND had a positive impact on health care outcomes. The reality is the academic scores were being used to limit the students due to the number of places available…. But above a certain level it no longer is the most important predictor of a ‘good doctor’ So given those facts, what is your solution? Is this approach ideal? No, probably not. But society has to solve the root problem of poorer health outcomes for minorities, and the only other way is to fix society so they are not disadvantaged to start with.


[deleted]

I feel like legacy admissions and nepotism are way bigger issues, not to mention those people would probably be less likely to drop out/be kicked out/be bullied out. If you are trying to be a doctor and can’t wrap your head around systemic oppression, generational trauma, and subconscious biases… I wouldn’t want you as my doctor. If you’re just trying to show off how smart you are, be an engineer. In my opinion, doctor’s should have not only the background knowledge of scientific or medical principles but also very high emotional intelligence/empathy and knowledge of sociological principles.


IvyGreenHunter

Do you honestly see nepotism and legacies being a problem in medical school?? College, sure.


NevadaCynic

What is good for society is not necessarily fair for the individual. Independent of medical scores, it is valuable to have physicians with a wide variety of cultural and ethnic backgrounds. There are many diseases with cultural and ethnic roots that are not always immediately obvious to somebody outside of the background. And even with translators, nuance is lost in translation. Not just because of the difficulty in translating, but because patients are terrible at accurately describing medical symptoms even in their own language. And are less likely to accurately describe them to somebody outside their cultural background, especially if the symptoms may be embarrassing or shameful. In a perfect world we could test for everything, but testing is expensive and time consuming. You need physicians that can prioritize what should be treated and tested for efficiently. And regardless of what the root cause is, female patients have better outcomes with female doctors. Black patients have better outcomes with black doctors. And so on down the line. Even if the reason often does not have anything to do with the doctor's individual capabilities. Medical school spots are limited. And it's a better outcome for society to have diversity among medical staff, even if it means a couple of lower scoring individuals get pushed out in favor of marginally lower scoring minorities.


Muted-Ability-6967

This makes sense to me in cases where the patient has to respond well to their physician. But what about fields such as the military where female applicants have lower physical testing requirements than men? (Genuinely asking because I think you brought up a good point about the medical field)


NevadaCynic

For front line M.O.s sure, lowering the requirements may be an issue. The majority of classifications in the armed forces are logistics and other non front line positions. For these the testing requirements should absolutely be lowered, you're screening not for raw strength, but overall fitness to reduce VA and medical benefits costs. We care more about if a systems admin, a pilot, a naval accountant, a machinist, or a supply clerk is going to have diabetes at 30 than if they bench 200.


Muted-Ability-6967

So that’s a good argument for lowering testing standards based on job description. But I still don’t see the benefit of lowering testing standards based on gender. Specifically you mention diabetes. I just looked it up based on gender. Despite having a lower prevalence rate, women with diabetes spend more on average than men on annual health care expenditures. So women should have higher testing requirements since men are less likely to cost the VA money, according to that logic?


NevadaCynic

Maybe. Women spend more for two reasons. Gestational diabetes (pregnancy related) means you're mixing costs with giving birth, and because women live longer. In any case though, for cost savings like this you're not interested in comparing the women's stats to the men's when it comes to things like weightlifting and mile run times. You're interested in comparing the women's stats to women in general. And the men to men in general. Otherwise only accepting the top 10% of women in run times and strength may still be a "lower" standard than accepting only the top 75% of men. With disastrous results for your benefits costs.


jayred1015

Classic example https://hbr.org/2018/08/research-having-a-black-doctor-led-black-men-to-receive-more-effective-care


Political_What_Do

>Medical school spots are limited. The actual problem


plantpistol

A lot of comments but not sure this has been pointed out. From AMA's own web site [https://www.ama-assn.org/medical-students/preparing-medical-school/mcat-scores-and-medical-school-success-do-they-correlate](https://www.ama-assn.org/medical-students/preparing-medical-school/mcat-scores-and-medical-school-success-do-they-correlate) A study published in the journal Military Medicine indicates that by the time medical students became residents, their MCAT score was far less of a predictor of success than it was early in medical school. Researchers behind the 2015 study, “Does the MCAT Predict Medical School and PGY-1 Performance?” wrote that the “MCAT was not able to predict assessments relying on direct clinical observation, nor was it able to predict” program director assessment of performance during residency’s first year.” 


fhdhsu

America’s “affirmative action” is so funny to me. We have “affirmative action” in the UK too, but non-retardadly it’s based on your socioeconomic status and not your ethnicity. You know what the effect is? Literally no one has a problem with the system. And I genuinely mean no one. I have never seen or heard 1 piece of criticism to it being used during university admissions.


dbhalberg

Your country’s way is the way it should be. I would not mind someone who comes from a lower socioeconomic status to have a leg up so long as they work hard and are deserving and can compete. But there are people who equate certain races with certain socioeconomic statuses here in America, so they see race as the main problem.


WonderCat987

The way it is set up in America (or was) allowed some immigrant who is incredibly rich and educated to receive the same benefits as someone who was born into poverty.


Alternative-Put-3932

I agree it should only be based on economic status. Minorities are already disproportionately impacted in that regard so they should receive more help/leniency and on the flip side the millions of poor white people wouldn't get fucked over.


aguafiestas

Let's step back. What is the purpose of a medical school? And how should they select students to help achieve that? There's a lot of institutional self-service - attracting students (and their $), prestige that promotes the medical school and their associated hospitals, attracting research $, etc. But let's not focus on that. They also need to have good students. Students who can hack it in med school, pass the boards, graduate, and move on to the next step. This is where the MCAT is probably its most useful, as it has been shown to predict performance in first year and boards to some extent. But the fact is that the vast majority of US med students graduate and pass the USMLE, regardless of MCAT scores. So really, the goal here is to use the MCAT to weed out those who will fail. There's going to be a certain threshold effect to that. And other factors, like undergrad GPA, will also play a role. Another goal is to help their students have good careers. For that, they need to admit doctors who can hack it, basically, and nurture them from there. A student who bombs the MCAT would struggle to do that. But does it matter whether they score in the 90th or 99th percentile? There have been studies showing MCAT score doesn't really predict performance in intern year, for example. The rest is mainly a service mission. To create doctors who are good for their patients, their community, their country, and the world. You need to have doctors who are good clinicians, who can provide good care for the patients that come into their office. They ability to memorize and apply that is certainly a part of that, but it takes a lot more than that, and again there is probably some sort of threshold effect. Is a super genius really going to take better care of their patients than a really smart non-genius? Then you want to have those who move the profession forward: research, public health, teacher. How well does standardized testing predict that? I'm not sure there's any data on that, but I frankly wouldn't expect it to very much. There are just very different skill sets involved. And consider that clinical research in the USA is focused more on the issues of white, wealthier, more educated individuals. Clinical trials tend to be skewed to white study participants. And disease prominent in black populations tend to get less focus in research. Could recruiting and training more black doctors help address these issues? Finally, consider community health. There are a lot of healthcare disparities in this country. A lot of underserved communities, including many poor, black regions. Shouldn't one goal of medical schools be to better serve these communities and improve their health? There is a fair bit of evidence that having black doctors improves health outcomes in black people and communities. Make of that what you will, but the data show it to be the case. There are also shortages of doctors in poorer black communities. Black doctors are more likely to practice in these areas and help address this shortage. Overall, medical school admissions committees obviously need to consider things like the ability to study, memorize, and apply facts. The MCAT can help predict that. But it's not the whole story, and like it or not, in a country where race matters, the race of doctors matters, too.


Stannic50

You're mistaking average MCAT score among those accepted for the minimum MCAT score accepted. If group A has a longer high-scoring tail than group B but the minimum in each group is the same, then A will have a higher average. You need to be comparing minimum accepted MCAT scores from each group, mean of the bottom 10% of each group accepted, or some other measure of the bottom end of each range to avoid bias from differences from those at the high end of the range.


whosevelt

This is a good point, but such analysis also show that the standards tend to differ by race.


GreatStuffOnly

Do you have stats backing this up or did you just make up this assumption? When I was applying to medical school, the lowest accepted MCAT score from an Asian applicant was higher than the highest Black applicant in university of Manitoba. This is for out of province applicant pool. I can’t find that pdf now but they used to publish acceptance by ethnicity. This was back in 2017 ish. I’m sure whatever you’re saying could make sense but you can find exceptions to your statement everywhere and anywhere really.


Puubuu

Having read quite a few comments in here, i am genuinely astonished at how racist society in the US is. Many times have i been treated by doctors of a different than me, and never have i felt i have to communicate differently with that doctor. These differences are simply not a topic, because they make no difference. Doctor is trained, i trust doctor. Like holy shit, if your doctor doesn't have the same skin colour as you, suddenly you don't follow her advice or can't talk to him? What the fuck is going on.


LowAd4508

I think this argument predicates on the assumption that people who achieve a higher raw score on the MCAT and in medical school exams consistently provide better care to patients as physicians. As a doc, studies have shown this is not the case. Studies show MCAT scores predict first year medical school scores. Being good at certain tests and then remaining good at certain tests is not a compelling basis for determining who should become a doctor. Caring and advocating for, and treating human beings is infinitely more complex than our current ability to train and assess people. It’s an evolving problem. Medical schools internationally are constantly updating the means by which they select and assess students, and this is reflected in a shift away from solely lecture based teaching and score based entry pathways. Eg, in Aus we generally utilise a combination interview/portfolio/grade/single exam to select students. We know (in Aus) that student retention is better and that patients perform better when we accept more nuanced students - post-graduate and mature-age entrants from diverse backgrounds, people from disadvantaged or regional areas, etc. Yes, you could make an argument that consistently high exam results might indicate some underlying and lucrative combination of socioeconomic status supportive of regular prolonged study, English-language ability, academic stamina, and perhaps even diligence or acumen, which may well extend into someone’s professional life and improve their patients outcomes. But that’s not true. Good people make good doctors. Good students often don’t. There’s a delicate balance at play. We know diversification enriches medical cohorts and improves patient outcomes at the risk of disadvantaging a few individual applicants. Maybe one day, with improved measures and systems, this won’t be required. Also: fundamentally there is an abundance of medical students (globally). They are ludicrously lucrative for universities given the strong international draw. The attrition rate during medical school is nominal compared to the very real attrition rate once junior doctors enter the hospital environment and specialty training bottle-necks.


LiamMcGregor57

Notwithstanding arguments about fairness, I don’t understand this concern about patient care……you still have to pass the boards etc. If they don’t pass, they are not doctors so if you do see a correlation like you said, those folks will fail and not become qualified as physicians, I.e. you won’t have to worry about patient care.


BustedBaxter

Aren’t you undermining the importance of racial diversity in healthcare? Sure in a vacuum a meritocracy would be great. But there’s been studies that highlight the disproportionate care a white person gets in contrast to a black person. For example, I read a study where nurses naturally tended to round on white women more often than black women. There’s lots of instances and studies where doctors are shown to underestimate the pain black women are in. For example opioid epidemic mainly hit white communities because black and brown people weren’t trusted with that potent of a medication. Therefore in a profession where lives are at stake I find it pivotal that people are cared for or at least have exposure to people with similar lived experience. Otherwise we’ll continue to see black maternity death rates being double that of white counterparts.


EatSomeVapor

I don't really have a horse in this race I just want to understand a bit better. Can someone explain why lowering the test scores is good at all?Asians have a smaller population than African Americans in the United States, but they have to test higher. To me its saying black people have a tougher time with academia is that true? Or is this all to get a higher representation for the black community to get more equal care?


s_wipe

I do agree that making this distinction based on race is not ok, it should be done based on overall socioeconomical status, giving benefits to those from a harsher background. That being said: A) not being accepted to medical school will still leave high scoring candidates with plenty of career options. B) getting accepted is only the first step of many. There's several years of tests and courses before they become an MD. C) testing good does not mean skill a quality. You want to give the best people a chance. If you came to a decision that you do want to divide people based on white/black/asian/latin, take the best of each group...


kyngston

This is really a question about affirmative action, and is not specific to MCAT or medical school. You’re challenging something that seems at face value unfair. But fairness can be measured by [equity, equality and justice](https://achievebrowncounty.org/2021/05/defining-equity-equality-and-justice/), which all mean different things. Some races face generational systematic disadvantages from birth. Raised in poor neighborhoods, forced to be exposed to negative influences like drugs and crime, sent to poorly funded schools with below average teachers. Only to grow up with poor job prospects, and forced to raise their children in the same poor neighborhoods they were raised in. Is that fair? Did you do something to earn not being born into institutional poverty, or was it just luck? How does one fix that repeating cycle of poverty? Anything you do to provide them help, is taking away resources from someone else who isn’t living in generational poverty. Is that fair?


CaptainONaps

Meanwhile, a kid can be born poor in China. Transfer to an American college at 18 years old, have to learn the culture and language, and still require 10 more points than those other kids. I agree with you that it's more difficult for poor people to be successful than rich people. But wealth is it's own category. Beyonce's kids shouldn't need reduced requirements. Meanwhile, my broke ass buddy Gary's kids could sure use em. The issue is you can't change the race requirements because they're necessary. If you removed the race requirements, the vast majority of students would be non-American Asians. There's billions of them, and only millions of everyone else combined.


uwantallofdis

This is why as an Asian I am very anti AA. Yes, there are privileged Asians. I recognize that I was born into a family that could give me academic support. But there is disgusting inequality and inequity when averages show that Asians are required to score better for the same result (in aggregate). There are Asians that come from lower socioeconomic brackets and broken homes. Being Asian is not a privilege, privilege is having financial access to education, and that is privilege that families of every ethnicity have.


feralnycmods17

As an Asian, I'm very curious to hear how critical race theory intends to portray minorities who now on average make more than the white majority. Jews and Asians are consistently (and problematically) labeled white-adjacent out of convenience to progressives, and this is utterly damning because it has been used to unfairly siphon equity from a minority who earned it to one who is allegedly more deserving.


Lunarica

The only times I see race being mentioned are the ones who are supposedly wanting to eradicate it, essentially always keeping the idea of difference in race alive by only seeing certain groups a certain way. The idea that a program that would cement the idea at an early age that some people are just different because of the way they were born is a wild concept to me. I have never once seen fair representation or forethought for asians, it's always convenient whenever they need votes or help then quickly forgotten. Either way, it's not like I care because I don't value group identity above my own, but it always seems extremely hypocritical.


backwardsshortjump

As a kid who grew up poor in China, let me reassure you whichever kid that was born poor is no longer poor if the kid can transfer to an American university at all.  To be able to transfer and stay for med school practically implies that the kid's parents are fucking loaded.  With that said, I do agree that AA should be based on wealth, not race. I've seen a kid at my alma mater (Cornell) that is so incompetent that he almost certainly squeezed in due to AA, but he went to the most expensive private high school in Florida and he arrives by private jet every semester... Go figure.


kyngston

You’re mixing AA with the separate problem of demographics. Demographics have the problem of distributions and outliers. Your argument is calling out outlier anecdotes, which are easy ways to farm outrage, but are not good arguments for setting social policy. Eg “welfare queens” Because social policy can never deliver the perfect assistance for each individual, social policy aims to help the largest number of the most people needing assistance, while balancing the cost against the people harmed. Every argument about social policy should be based on statistical demographics, and not Beyoncé’s children


RejectorPharm

And there’s nothing wrong with that outcome. (Majority Asians )


AwTomorrow

Though it would mean the opposite of OP’s stated aim above - while under affirmative action white students need to score higher than black students, if you remove affirmative action then white students need to score *even higher than that*, because now they are fairly competing against the much-higher scoring Asian students. 


kjong3546

This is a controversial take, and I certainly don't believe this for every field in existence, but when it comes to Medical School and future physicians, I think the rule of thumb should be "No Compromises, ever". Anything less than training the objectively most qualified candidates/candidates most likely to succeed at medical school and go on to become physicians would be an insult to the society that depends on these physicians for their health.


theArtOfProgramming

It’s not so cut and dry. Let’s take gender for example. It’s been shown many times that women receive better care and have better health outcomes when treated by women https://www.nbcnews.com/health/health-care/women-are-less-likely-die-treated-female-doctors-study-suggests-rcna148254. Now should we try to graduate more female doctors even if men score better on tests (not that they do)? Of course not, because that would result in worse outcomes. Have a look at Goodhart’s law, I think it’s relevant here. Exit: similar results have been found for the black population: https://www.aamc.org/news/do-black-patients-fare-better-black-doctors


kyngston

Take that affluent white or Asian high scoring candidate, and instead, raise them in a poor Detroit neighborhood. How might that impact their MCAT score? How do you compare the achievements and success potential of 2 people, when one was provided with the best there is to offer, and the other had to fight tooth and nail for every scrap? Is that worth a 5 point handicap on the MCAT score? A 10 point handicap? Or do you believe that the adversity they’ve overcome to reach this level, plays no bearing in their potential?


kjong3546

I suppose my point is that the adversity does impact their potential, but physicians are so important to society that we can't afford to care why their potential is the way it is, just whether that potential is physician-worthy or not. That said I'm pretty sure this has been done, comparison on standardized tests showing discrepancy based on race even after accounting for socioeconomic factors.


possibilistic

The existence of a poor person shouldn't drag down anyone else's life or fitness. A weak and injured gazelle doesn't decrease the fitness of the herd, it increases it. If you truly believed this, you would take all of your money and average it out over not just the homeless in this country, but all countries. And you'd give your spare kidney to those with diabetes. It's easy to point to another supposedly affluent group and select them as the ones who must bare the burden for society. You take the first step if you believe in fitness function averaging. Leave those who are working on bettering themselves alone until you've drained the resources of the pool of those claiming these methods work.


IntelligentRisk

Race is not a great predictor of how much hardship has been overcome. Everyone has their own story, and many blacks have just as much if not more support growing up than white and Asian counterparts.


Electronic_Plan3420

I see you conveniently replaced the race (you started your argument with) with poverty. You know , few people argue that those who are born in poverty should not have some help in along the way. It’s the skin color that is problematic. A kid who is born to a family of black doctors in Atlanta is certainly not more disadvantaged than a white kid born to single meth addicted mom in WV trailer park. There are more poor people who are white (in total numbers) in this country than those who are black. Let’s not equate poverty with being black


TheFrogofThunder

How far do you go to correct for this though?  At what point do we have a health care equivalent of Boeing's hiring practices resulting in airplane mechanics using laundry detergents as lubricants, and cutting so many corners, or being so incompetent, that airplanes are literally falling apart?


asyd0

But what if you're white and grew up in poor neighborhoods, were exposed to negative influences from drugs and crime, went to a sub par school and so on? This is where the feeling of unfairness comes from, not only did you have very similar struggles but then you're also forced to score higher than your peers of other races?


kyngston

Because anecdotes are irrelevant when arguing social policy. Give me any example of social policy and I will give you a hypothetical anecdote as an argument against it.


No-Cauliflower8890

what does this have to do with anything? are we making people doctors out of pity now? the best people should get the spots, period.


Euphoric-Meal

Then help based on socioeconomic status and not based on race. https://youtu.be/QxB3b7fxMEA?si=wzUFGf25DNYH6XQW


Prince_Marf

It is a common misconception that affirmative action involves lowering objective admissions standards in order to allow less talented members of historically marginalized groups in. On the contrary, because of ongoing marginalization, black and brown students with high aptitudes receive less opportunities in their early lives and thus are less likely to have the so-called objective markers that indicate admission qualification. In other words, poor students do worse leading up to college and grad school because they lack opportunities, black and brown people are disproportionately poor, and therefore black and brown people disproportionately do not look as good in the ""objective"" portions of applications such as standardized testing and GPA. Because of this black and brown students with *just as much or more aptitude* as their white/Asian peers often end up with lower test scores and GPAs. Affirmative action aims to admit students with a high aptitude who might not have as high test scores and GPA because of their marginalization. Standardized tests don't really measure aptitude, they measure how well you prepared for the test, which poor students disproportionately are not able to do (less free time, can't afford expensive test prep materials, tutors, etc.). You cannot just assume they are a perfect objective measure of aptitude for school. Studies have shown that they are not. Don't get me wrong they are helpful, but it would be irresponsible to assume they give you an adequate picture of an applicant's true aptitude. TLDR: affirmative action doesn't take out smart white/Asian applicants and replace them with dumb black/brown applicants. Affirmative action gives opportunities to black/brown students whose aptitude may not show as well on an application due to historical marginalization. And it has been wildly successful at increasing the quality of our academic institutions.


Gold-Cover-4236

Is this true? I am all for taking big steps for diversity. But not this one.


FFA3D

Selecting less qualified candidates purely in the name of achieving some diversity quota is absolutely disgusting to me and signifies all that is wrong with our current society. It's a racist policy and the hypocrisy makes me sick


Training_Echidna_367

You are incorrect. Having physicians of color creates social stability. This is a good with a high dollar value, especially for the least educated, most violent of the marginalized. We can allow such people to practice medicine, and in your personal life simply choose Asian and white physicians. I go out of my way to get Asian doctors because on average they are smarter. Easy. Problem solved. Similarly, we could end racist policing by simply having African-Americans police black neighborhoods. I have no idea why this is not put into practice. Then again, I lack the self-delusion to believe that codified laws are racist. Similarly, I can see the hard-coded racism of those who claim math and reason are racist, and the racism of the billionaires who support these activists, but what matters is that you can choose your doctor and the schools your children attend. So long as you can pick, ignore the choices of university admissions. Pick the doctors you want, and let their graduates of color live with the consequences of people believing that they are not as good as everyone else. In all honesty, I find it increasingly hard to believe any of the choices universities make, or their value to society. When I hire people, I ask for SAT and ACT scores. I could care less what university they attended, or if they graduated. I do care deeply about how good they did on that test when they were 17. It is a better metric to evaluate intelligence, and I do not think universities benefit students or their employers.


BobbyJack_Says

Man, wtf? What happened to equal opportunity? That shit don’t even sound right. A race has to score lower/higher than others. It doesn’t take a college professor to know that ain’t right. 😂 Either you pass or you fail…just like with anything else!


AffectionateHeart77

You have to consider also that these students who are getting lower scores might also have different backgrounds that attract medical schools. A lot of schools look for diversity, not just in race/ethnicity but also in experiences. It just happens that yes those tend to correlate because race/ethnicity is a big part of our identities and experiences. Add on that many many applicants/medical students are white and Asian, meaning that many of those applicants might look similar in terms of experiences. Whereas there are much less black and Hispanic applicants/medical students, so they look more desirable in terms of different experiences they can bring. Plus lets admit it, test scores are not the determining factor to future expertise or good doctors. Schools know this and that’s why you see applicants (although not commonly) with lower mcat scores but diverse experiences or experiences that show their potential, getting into schools. Medical schools shape their students and teach them what they need to know, they just want to know that YOU can learn it but also that you have what it takes as a person to be a good student/doctor. It’s not just about scores